There is no Proof that CO2 is Causing Global Warming

This is just one of dozens of responses to common climate change denial arguments, which can all be found at How to Talk to a Climate Sceptic.


Objection:

Correlation is not proof of causation. There is simply no proof that CO2 is the cause of the current warming.

Answer:

There is no "proof" in science, that is a property of mathematics. In science, one must look at the balance of evidence and formulate theories that can explain this evidence. Where possible scientists make predictions and design experiments to confirm, modify or contradict their theories and must modify these theories as new information comes in.

In the case of the theory of Anthropogenic Global Warming, what we do have is a theory (first conceived over 100 years ago) that is based on well established laws of physics, it is consistent with mountains of observation and data, both contemporary and historical, and it is supported by very sophisticated and refined Global Climate Models that can successfully reproduce the climate's behaviour over the last century.

Given the lack of any extra planet Earths and a few really large time machines, it is simply impossible to do any better than this.

Aside: It is usually interesting to ask just what observations or evidence your skeptic would consider "proof" that Global Warming is indeed caused by the rising CO2 levels. Don't be surprised if it an impossible request!


This is just one of dozens of responses to common climate change denial arguments, which can all be found at How to Talk to a Climate Sceptic.


"There is no Proof that CO2 is Causing Global Warming" was first published here, where you can still find the original comment thread. This updated version is also posted on the Grist website, where additional comments can be found, though the author, Coby Beck, does not monitor or respond there.

More like this

Co2 does not correlate to temperature over the last 150 years,or the last 1000 years,or the last half-million years[Vostok ice cores].There were times in the past when co2 was 5 times or even 10 times more abundant in the atmosphere,and the earths temperture was far lower than it is today.Al Gore's attempt to correlate them was nothing less than scientific fraud as it is well known that co2 follows temprature,and not the other way around which is what the AGW theory requires.

By Tim Wells (not verified) on 19 Aug 2008 #permalink

none of the links in this article take me to the evidence that CO2 is causing the warming. I keep asking everywhere- what is the evidence that CO2 is to blame? Nobody has an answer.
So, Mr Coby, since you have set yourself up as an expert and spokesman on this issue, I want to hear from you.
What... in your words... is the evidence that C02 is causing the global warming?

By daddy dave (not verified) on 15 Oct 2008 #permalink

daddy dave:
Given the absorption spectrum for CO2, how is it possible for increasing atmospheric CO2 not to increase the mean temperature of Earth?

By Richard Simons (not verified) on 20 Oct 2008 #permalink

Richard Simons:
Co2 is a greenhouse gas ofcourse,and it does add a small amount of warming.But it has not been shown that the .7C rise in the last century is attributable to CO2.It is simply not a strong enough greenhouse gas.CO2's radiative effect is,as you know,logarithmic and so has less of an effect for each given volume that is put into the atmosphere.The majority of the radiative effect of CO2 is already done as the absoption wavelengths are now close to saturation.

timwells,

"CO2's radiative effect is,as you know,logarithmic and so has less of an effect for each given volume that is put into the atmosphere."

Yes, we know that. And you know we know it. So why bring it up? The CO2 forcing estimates which are used to attribute warming to CO2 already include this effect. The forcing is still large enough. "Increases slower than linearly" doesn't mean "negligible".

I detect a pattern of errors here. Climate scientists know that CO2 forcing is logarithmic in concentration. They know that temperature rises can cause CO2 increases. And they know that CO2 levels have been larger in the past. Your assumption seems to be that scientists don't know any of these well known facts, or else they ignore them because Al Gore told them to.

In reality, as coby pointed out, these facts don't prove what you want them to prove. But upon having this pointed out, you simply ignore those points and move on to new "arguments" that scientists are supposedly unaware of.

After being corrected once, you should have reconsidered your "global warming is disproven by this well known fact unknown to scientists" strategy, but you stuck to it. It shows that you'd rather quote-mine arguments off skeptic websites than think about the actual science involved.

By Ambitwistor (not verified) on 21 Oct 2008 #permalink

Mr Ambitwistor - I'm very happy debate the science involved.

TimWells suggests that of the 0.7 degrees rise in the last century, the portion of this that can be attributed directly to CO2 is small (only that which occurred between the late 1970s to the late 1990s can possibly be due to it for starters). I have asked this question myself on this site

http://scienceblogs.com/illconsidered/2008/09/arctic-sea-ice-recovered-…

inertia is only answer I have had so far, an explanation I am not convinced by, though that particular subject is no doubt complex.

At least those commenters gave me a vaguely plausible explanation. However, your answer to TimWells appeared to be:

-state that TW said something you agree with (real zinger that one!).
-then put words in his mouth that he clearly didnt mean ie. " Increases slower than linearly" doesn't mean "negligible". We all know this.
-then say that TW makes the claim that scientists do not know a series of three facts despite the fact that, erm, TW explicitly used the phrase as you know in his post with regard to one of the facts, explicitly stated in his post another of these facts (that CO2 does add a small amount of warming), and didnt mention anything in connection with the third.
-I could go on..

There are no substantitive points in your post. There is only one person in this twosome that I think ought to think about the actual science involved.

In many years, our teachers, tech us that CO2 is the main cause of global warming, now it doesn't via this post, who can we believe now?

The warming trend could be due to instrument changes over the years. I have read some articles from Yale recently showing how changes in instrumentation can fowl up the data.

By William Pinn (not verified) on 13 Apr 2009 #permalink

William Pinn -

Would you care to share this article with us? Because I'm willing to bet that you're reading more into it than should be.

Well, where to start? There are obviously many facts put together and interpreted for the AGW to be forwarded as a real current event and many used by detractors. Obviously, as well, for those who study it, discussion here will not sway one or another from their position. So let me ask a broader question.
What series of events, data, etc. over what period of time would give you pause to reflect on the reality of either side?
One example: IPCC has put forth a temperature trend forecast with min and max expected range over the next several years. What if the actual was above this range? What if the actual was below this range, or trending in the opposite direction for some length of time?

I look forward to your thoughts.

Paul -

I think you're actually asking for several different answers. I'll just address the main one (I think) that you were driving at, that human industrial activity is driving global temperatures upward.

For me, personally, it would probably take a long-term negative temperature trend (or a stable trend) in the absence of significant 'external' effects (i.e. a sudden increase in volcano activity that might dim the sun) as atmospheric greenhouse gas concentrations continue to rise. Long-term is fuzzy, but probably around a 30 year trend.

I'm sure there's more cases that might convince me otherwise, or specific things that I might become more doubtful of. That being said, the evidence for anthropogenic global warming is pretty broad, so there's unlikely to be a 'magic bullet,' so to speak, that would do it for me.

Adam,
You have it correct.
I am asking for individuals thoughts on what it would require for them pause to (re)consider what is happening, from either side, or those unsure.
Thanks for your input.

Adam,the evidence for recent warming is "pretty broad",but the evidence that humans have caused this warming is negligible.

By tim wells (not verified) on 14 Apr 2009 #permalink

Adam....

That was a great link you posted and I hope Tim Wells takes a good look at it and notices that it is filled with "Facts" and "Myths".

Hey Tim, check out this statement in Fact #1.....

"increased concentrations have contributed to the recent warming and probably most of the warming over the last 50 years."

So "probably" is a fact, or is it "probably" a fact?

Here's a good one in Fact #3.....

"These temperature increases are likely to result in an increased frequency and severity of weather events such as heatwaves, storms and flooding."

So "likely" is a fact, or is it "likely" a fact?

Here's a great one in Fact #6.....

"Computer models cannot predict the future exactly, due to the large number of uncertainties involved."

Oh wait, that actually is a fact. Nevermind.

Coby......

Interesting. You provide a link to a post that tells how to respond to a statement that I never made.

Here's the point you missed. If someone is to list AGW as "fact", then perhaps it would be best to be honest and use "probable fact" or "most likely fact" or perhaps "uncertain fact".

C'mon, it's almost as if you were on another planet....oh wait, you provided that link as well.

Betula -

You're entire qualm with the theory is that people use imperfect language? Or is imperfect language just another shadowy player in the double-secret global envirofascist conspiracy?

"You're entire qualm with the theory is that people use imperfect language?"

No Adam, my partial qualm with the theory is that it is a theory treated as fact i.e., people using imperfect reasoning.

Though I do admit I have a problem with people using hyperbole to justify a chimera.

Causality... Is there any evidence that the rise of temperature could cause the rise CO2 level?

Sunburn: Yes. Every person who's taken an intro chemistry course knows about the solubility of gases in liquids. The higher the temperature, the less gas (in this case CO2) can be dissolved in a liquid (in this case water; the oceans). That is to say that if the temperature goes up, the atmospheric CO2 must go up as well, because it is taken out of the ocean. This is one of the reasons we keep soda (and other carbonated beverages) cold!

The inverse relationship between temperature and solubility of a gas in a liquid has been known for a long time. It astounds me that so many 'scientists' somehow forget this fact.

Also, to clarify, I do not believe that the current rise in CO2 is a direct effect of the rise in temperature; I'm well aware that we've been adding lots of CO2 to the atmosphere. However, it is important to keep the facts of what I have said in mind when going over the ice core data and the old 'CO2 lags not leads' argument.

Chris:
"It astounds me that so many 'scientists' somehow forget this fact."

I trust you have a reason to think this other than some anonymous commenter calling himself "sunburn" forgetting that fact? Perhaps you can share it, or perhaps you are completely mistaken that any scientist having anything to do with earth, ocean or atmospheric sciences is unaware of that.

The main reason I bring this up is the importance of that fact (solubility of a gas in a liquid) is handwaved away when discussing the ice core data and the 'lags not leads' argument. The simplest explanation for the data we see in the ice cores is that temperature rose, for some reason, and thus CO2 went up as a direct result. Then temperature took a dive, and CO2 took another few hundred years to start dropping too. This shouldn't be surprising to anybody. It is also NOT in any way indicative of a strong CO2 feedback. All we see is what we'd expect if we considered some trace gas in contact with a liquid and the temperature of the system in flux (for some external reason).

Chris, it is not hand-waved away by anyone familiar with the science. I mention it myself directly in many places. Check this article for an explanation of the ice core data. It has been studied extensively and continues to be.

Basically the driver of these cycles are very small forcings from orbital changes (Milankovich cycles), but the feedback from land ice melting and greenhouse gases coming in and out of the ocean (as you describe) are the two most significant contributors to the total eventual swings in temperature. These effects can be and have been quantified, orbital forcing alone is no where near enough to cause ~10oC temperature swings.

"These effects... have been quantified..."
So Coby please show us the math.

By timobrienwells (not verified) on 27 Apr 2009 #permalink

Coby,

You have once again made an unfounded statement. We know that CO2 is not a major driver of warming. As I stated in the other thread, the long term high resolution record (ice cores) shows that temperature rises and 800-2500 years later the CO2 starts rising. Further, the record shows that CO2 cannot be a driver since peak warming happens before peak CO2 in the record.

This does not mean that CO2 causes no warming but rather the climate driver, which you term as minor, causes more warming than CO2. Please point to a study that can actually show what warming is due to CO2 and what is due to the rebound from the LIA? Oh, and please, no circular logic, i.e. please do not point to your own articles as proof of this article.

Interesting thread. Seems like some real science would help here. There appears to be some debate about whether or not CO2 levels demonstrably cause warming. here are a few quotes:

Says Dr. Syun-Ichi Akasofu, the director of the International Arctic Research Center, "[man-made] CO2 began to increase exponentially in about 1940 but the temperature actually began to decrease and continued 'til [sic] 1975. We cannot say that CO2 and temperature go together." Dr. Nir Shaviv of the Institute of Physics, University of Jerusalem states that "there were periods in Earth history when we had three times as much CO2 as we have today or ten times as much CO2 as we have today and if CO2 has a large effect on climate we would see it in the temperature reconstruction."

Dr. Piers Corbyn, lead climate forecaster for Weather Action (England) said, "None of the major climate changes in the last 1000 years can be explained by CO2." Dr. Patrick Michaels from the University of Virginia and IPCC author states that "Anyone who goes around and says that CO2 is responsible for most of the warming of the 20th century hasn't looked at the basic numbers."

Dr. Ian Clark a leading paleo-climatologist from the University of Ottawa says, "If we look at climate in the geological timeframe we would never suspect CO2 as a major climate driver. You can't say that CO2 will drive climate. It certainly never did in the past."

To Paul, re post number 11.

This is a good question, ALL of the IPCC predictions or guesses have so far failed miserably for example;

The IPCC prediction of CO2 levels is above current levels
The IPCC prediction of Temps is above current levels
The IPCC prediction of sea levels is above current levels
Ice extent is above IPCC predictions and the list goes on.

It is obvious that CO2 is not the so called climate driver thier TAX scheme is relying on and something else is actually driving the climate. If it is daytime and you go outside you will be able to see what this something else is.

Just look up in the sky, but no people like Adam will continue to say CO2 drives the climate and a 0.whatever rise in temp over the past 50 odd years is proof positive.

I found this and thought it would be good to post here, it explains the fallacy of the computer models and the over exaggeration of CO2 effects on climate by the IPCC.

Of course Adam Et al will simply dismiss this story out of hand as they are blinded by thier faith. But to the more open minded readers out there you will find i very interesting reading. This story hoses down all the hyperbole of alarmism and puts it all into perspective.

http://scienceandpublicpolicy.org/scarewatch/dangerous_climate_change_i…

Cheers

Crakar

Crakar, do you understand what it means to be skeptical? If so, please let us know how you verified this remarkable claim from the first link you offered:
Nature is one of many âscientificâ journals that have openly declared an editorial prejudice in favor of a frankly alarmist viewpoint on the climate. In short, Nature adamantly refuses to publish any paper suggesting â however compelling the evidence and arguments â that anthropogenic âglobal warmingâ will not be as significant as the UNâs climate panel suggests. Natureâs selection process is, therefore, openly prejudiced ab initio. In reality, Nature is now a religious rather than a scientific journal.

I am quite certain your professed skeptical nature is just an excuse to swallow whole any ridiculous diatribe you find that confirms your existing bias against believing in AGW. And it was a diatribe, it was a stream of colourful and emphatic assertions without a single attempt at substantiation.

Monkton is the best you`ve got?

So rather than respond to any of the scientific content of the story you merely respond to to mans critcism of the nature journals publication process.

I take it the scientific content of the story has some merit then?

Maybe you have a better explanation as to why the IPCC's predicted values for CO2 and temp levels are completely wrong in an upward direction after only 8 years. Perhaps as he says they have over exaggerated things. Do you have a better explanation for why the models are wrong?

Sorry Crakar, I thought I made it clear I didn`t think there was any scientific content. Now how about addressing my question as to how you applied your skeptical nature to what this fellow wrote? It should be easy to verify if Nature Magazine has openly declared a prejudice as Monkton claims.

Since I am already positive you have no real skepticism and swallow whole any article that confirms your bias, perhaps we can move on to why you trust Lord Monkton so much that you do not ask for even the most basic substantiation from him?

Coby,

How about addressing the issues I raised?

This seems like a good time and place for me to point out that persons in favor of AGW too often want to have it both ways when it comes to climate sensitivity. They claim that our current rise in temperature is due mostly to CO2. Ok, fine. But when it is pointed out that the climate sensitivities used by the IPCC in their projections would, if they were correct, predict far more warming that we've seen from the current rise in CO2. This argument is 'refuted' by claiming that there's warming in the pipeline. Alright, fine. Why then do we see so many graphs that show CO2 and temperature rising nice and even with each other from the 1940s on? Why are these graphs and the correlations they imply thrown around if there's some delay in warming caused by 'warming in the pipeline?' You cant have your climate sensitivity and eat it too. Either the current warming is due to CO2, and climate sensitivity is lower than the IPCC says (on the order of 1.5, instead of 3 or 4 or more) or the current warming is due in some part to CO2, but could have been from some other source as it was finally emerging from the so called pipeline. The problem is that this pipeline theory can step in and 'save' any broken theory simply by saying 'well, the warming we predicted... it's coming later.' Might this be the case? Yes, of course. But I have yet to see an argument which embraces the 'pipeline' for CO2 and also admits that perhaps some or all our current warming is simply heat from a different 'pipeline' emerging.

Vernon,
You wrote:
"the long term high resolution record (ice cores) shows that temperature rises and 800-2500 years later the CO2 starts rising."

I have not seen this range before, can you please substantiate it? The numbers I am familiar with are 600-800+/-200yrs. I do recall that the lag may be even higher at particular times in the record, but if you have one of these times in mind. please specify.

"Further, the record shows that CO2 cannot be a driver since peak warming happens before peak CO2 in the record."

This is simply not a valid conclusion. It only means that CO2 can not be the only driver, which is non-controversial.

"This does not mean that CO2 causes no warming but rather the climate driver, which you term as minor, causes more warming than CO2."

Please share with us how you have derived this, and perhaps some numbers. How much more? What drivers? How many W/m^2 are we talking about? I am happy also to be pointed to a study, preferably with a quoted passage, that supports your position.

"Please point to a study that can actually show what warming is due to CO2 and what is due to the rebound from the LIA?"

Try this: Meehl, G.A., W.M. Washington, C.A. Ammann, J.M. Arblaster, T.M.L. Wigleym and C. Tebaldi (2004). "Combinations of Natural and Anthropogenic Forcings in Twentieth-Century Climate". Journal of Climate 17: 3721-3727. This is the cited study for a very good graphic you can look at here. Can you please be more specific about what you mean by "rebound from the LIA". Why do you think the climate system is like a rubber band that will always return to a pre-LIA state?

"Oh, and please, no circular logic, i.e. please do not point to your own articles as proof of this article."

That is not what circular logic means. If I have answered one of you points in another post, I will point you to it, nothing wrong with that. Why should I type out the same explanations over and over?

Coby the only thing you made clear was to attack the mans thoughts on the Nature magazine. Fine attack him all you like. The question still remains about the accuracy of the IPCC modelled results. But please continue to pretend this is not the issue up for debate and we shall avoid it by having a go at Monkton.

Well done Coby

Crakar, I did not attack `Monkton's "thoughts" on Nature magazine, I asked you if you verified that claim, as a skeptic would when confronted with a very strong assertion like that. Are you a skeptic or a denialist? Do you verify what you read or just knee jerk believe/disbelieve according to your preconceptions?

Did Nature Magazine "openly declare[] an editorial prejudice in favor of a frankly alarmist viewpoint on the climate."? Did they "adamantly refuse[] to publish any paper suggesting â however compelling the evidence and arguments â that anthropogenic âglobal warmingâ will not be as significant as the UNâs climate panel suggests."?

Do you care if Monkton lies about these things or not?

I think Monktons assertions are that nature only publish one side of the story. Beyond that who gives a toss. Now onto more important issues. He claims that temp increase through AGW will be less than 1C.

How do you respond to that? Or are you still avioding the issue?

I have read and investigated enough of Monkton`s writings to know that he does merit any serious person`s time. You are avoiding my question to you. Monkton does not just say Nature is onesided, that is standard subjective FUD. He makes an easily verifiable claim that Nature has made an open declaration.

If you can not show us where this alledged declaration is, why should we trust a liar? If you have not tried to find out, or don`t care if it is true, how can you claim to be skeptical and serious about this issue?

Coby,

You have done everything humanly possible to avoid discussing the scientific nature of the article.

Maybe you could supply a list of websites that you consider to be worthy of using as a source of information. This way you will not have to embarass yourself again like this.

Coby,
I sympathise with you and sincerely feel the pain you feel in having to answer to all the nonsense. I also feel the pain deniers feel by having to live with themselves 24 hours a day, 7 days a week.

I wish I was wrong but I feel more and more like humanity has to learn this lesson the hard way. If we survive, we'll probably be wiser. Or maybe not. Pity we have to bring so many other living beings down with us. But one thing I'm surer and surer about: the average human mind cannot and does not want to grasp the epochal changes about to befall on us so we'll cling to our destructive lifestyle until it'll be too late. Given the choice between feasting today and starving tomorrow or fasting for a day and having just enough for tomorrow, we choose the former. So be it.

Cheers and keep up the good work
(but take it easy, it's useless anyway)
Alex.

This is a must read. If you dare!

IPCC reviewer and climate researcher and chemist Dr. Vincent Gray of New Zealand is an expert reviewer on every single draft of the IPCC reports going back to 1990 and author of The Greenhouse Delusion: A Critique of "Climate Change 2001. Dr. Gray's research is featured on page 155 of the 2009 edition of the 255-page "U.S. Senate Minority Report Update: More Than 700 International Scientists Dissent Over Man-Made Global Warming Claims"

http://climatedepot.com/a/611/UN-IPCC-Scientist-No-convincing-scientifi…

At least admit that there is no consensus.

The dirty little secret is that the CO2 connection to global warming is a backdoor attempt to stop any form of new fossil fuel energy development by enviromentalist wackos.

Yes, Steve, you are on to us! Greenpeace has infiltrated the highest levels of governments around the globe, all major research institutions, the UN and defeated the likes of Exxon Mobile as they try to bring you the Truth.

Now stop letting the world know at once or my masters will have you shoveling coal on the Sea Shepard faster than you can say "All hail Al Gore"

Coby,

Ok, studies that show the various points that you are unaware of, but then I was not staying current on this either so I was a little off too:

Ahn and Brook (2007) atmospheric CO2 is strongly correlated with the Antarctic isotopic temperature proxy with an average time lag of 720 ± 370 yr (mean ± 1Ï) during the time interval studied.

Siegenthaler et al (2005) lag of CO2 to temperature of 800, 1600, and 2800 years.

Caillon et al. (2003) 800 +/- 200 years for termination III.

Monnin et all (2001) Science 291, 112; found a lag of 800 +/- 600 years for termination I.

Fischer et al. (1999) 600 +/- 400 years for the past three transitions.

Mudelsee (2001) Vostok's CO2 record lag behind those of its air-temperature record by 1.3 +/- 1.0 ka

Toggweiler (2007) CO2 was responsible for less than 30 percent of Antarctic warming during the glacial terminations.

Hogg (2008) temporal response is asymmetric due to a large outgassing of CO2 produced by warming. This effect is a limited positive feedback triggered by the insolation cycle.

Watson et al (2006) The observed very close link between Antarctic temperatures and atmospheric CO2 could then be explained as a natural consequence of the connection between the airâsea buoyancy flux and upwelling in the Southern Ocean, if slower ventilation of the Southern Ocean led to lower atmospheric CO2.

Oh and I did read the study you referenced and I have to say I believe the author when he says âForcing uncertainties, however, admit a quite wide range of sensitivity possibilities.â I say this because none of the studies he referenced was a study that shows what the composition and quantity of the aerosols were. I believe Hansen had it right when he said that the amount and type of aerosols is unknown both now and in the past. Anyone, he said, that placed a value for aerosols was pulling numbers out of a hat.

And he still does not show what warming is due to the LIA rebound and what is due to GHGs.

Now as to why I say rebound from the LIA, Hunt (2006) shows

While a number of characteristics of the MWP and the LIA could have been partially caused by natural processes within the climatic system, the inability of the model to reproduce the observed hemispheric mean temperature anomalies associated with these events indicates that external forcing must have been involved. Essentially the unforced climatic system is unable to sustain the generation of long-term climatic anomalies.

It stands to reason that whatever external forcing that caused the cooling had stopped and the temperature was moving back to the higher equilibrium in the absence of said forcing.

Steve -

I can't tell if you're trying to joke/be sarcastic by referencing Inhofe's list.

So, just to be safe, if you're being serious, it's complete and utter bollocks.
http://greenfyre.wordpress.com/denier-vs-skeptic/denier-myths-debunked/…

If you're being sarcastic, please disregard this comment.

Vernon -

I'm not sure what you're trying to accomplish with referencing these studies. No one is denying that CO2 increases lag temperature increases at these points in time.

No sarcasm intended on the link. So you are disregarding Dr. Vincent Gray and the 700 other scientists who dispute man-made global warming? Guess they are all in bed with "Big Oil" (sarcasm intended). Your "consensus" is slowly but surely eroding away.

Adam,

If you read Coby's post you would have your answer.

Steve -

No, I'm definitely not saying that they are being paid off. I'm just saying I don't care what the opinion of a TV weathermen is.
http://www.ethicsdaily.com/news.php?viewStoryPrinter=14084

One of the listed prominent scientists is Chris Allen, who holds no college degree, believes in creationism and belongs to a Southern Baptist church.
Allen is a weatherman at the FOX-affiliated TV station in Bowling Green, Ky.

Examples of this abound. Be a real skeptic, Steve. Don't just blithely, uncritically accept everything that supports your preconception.

Steve -

Why do you accept the judgment of a nuclear physicist over a climatologist when it comes to studying the climate system. Why disregard the opinion of experts in favor of nonexperts? Do you also go see an endocrinologist if you break your ankle? An auto-mechanic to repair your TV?

Dr. Lycklama might be an excellent nuclear physicist, but I have doubts about his ability to discuss global warming, especially since he manages to seriously suggest that Inhofe's list and the Oregon Petition are valid indications of 'non-consensus'.

http://www.nizkor.org/features/fallacies/appeal-to-authority.html
This fallacy is committed when the person in question is not a legitimate authority on the subject. More formally, if person A is not qualified to make reliable claims in subject S, then the argument will be fallacious.

So rather than addressing their arguments against, you perfer to discredit all of them in one fell swoop. So lets go down that road. How many of the "scientists" associated with the IPCC are truly qualified and legit? Lets put the qualifications of those scientists up against the qualifications of scientists on the other side. There are a lot of very smart, well qualified individuals who don't believe that CO2 has anywhere near the influence on climate that the proponents of purport. I guess James Hansen is smarter than all of them.

Adam,

So in your mind, only climatologists are qualified to offer an opinion on global warming? Biologists, chemists, geologists, astrophysicists, nuclear physicists, etc. are not qualified and have nothing to contribute to the subject? Is Al Gore a climatologist?

I suppose Dr. Vincent Gray, IPCC reviewer, is also unqualified, at least when he disagrees.

Coby,

So, when are you going to address my argument. Do you believe that ignoring this is going to make it go away?

I have addressed your questions, so why to you not want to discuss the science and why it does not support your position.

(alos posted on the incorrect thread http://scienceblogs.com/illconsidered/2006/02/co2-lags-not-leads.php#c1… )
Steve,

Did you actually read that article? From the top of its second page:

"He stresses that he has little problem with the basic scientific understanding of climate change. Itâs just that, if progress is to be made in debates on how to respond to that knowledge, they need to be opened up to other disciplines, from the arts and humanities, for example - and to good old-fashioned politics and ideologies."

If so, what is your point?

My point is, that he is willing to listen to other disciplines. And that others should be willing to as well. He also doesn't seem to have much faith in the IPCC which you hold so dearly as the final answer on all that has to do with global warming.

Steve -

I'm very happy to listen to other disciplines. I agree that not enough has been invested in studying the sociological effects. I don't believe anyone is actually suggesting that we ignore everyone except climatologists.

What I don't accept, however, is denialists who recycle talking points that have long been debunked (no consensus, stopped in 1998, CO2 has no influence on temperatures, it's the sun, etc.). Mike Hulme is not one of these people.

Additionally, I see very little value in lists of dubious quality (to put it nicely) of 'scientists' like Inhofe's who reject the consensus opinion. This list is clearly a load of bollocks, as even the most cursory investigation will reveal.

How many of the "scientists" associated with the IPCC are truly qualified and legit?

I await your investigation. Please report back when complete.

There are a lot of very smart, well qualified individuals who don't believe that CO2 has anywhere near the influence on climate that the proponents of purport.

I see no evidence of this. Just saying it doesn't make it true. If you're referring to Inhofe's list, we've already established why its not valid. Otherwise, please provide evidence.

So in your mind, only climatologists are qualified to offer an opinion on global warming? Biologists, chemists, geologists, astrophysicists, nuclear physicists, etc. are not qualified and have nothing to contribute to the subject?

No, biologists can offer insight into how increasing temperatures affect animal/plant populations. Geologists can offer historical perspectives. Everyone can contribute in their field as relates to global warming. But, I see no reason to take their judgment on the basic science of climatology more seriously or as seriously as a climatologist. Just how I don't ask climatologists to tell me about neutrinos and quarks.

I suppose Dr. Vincent Gray, IPCC reviewer, is also unqualified, at least when he disagrees.

Vincent Gray has a PHD in physical chemistry. I'm sure he's an excellent chemist, but, as stated above, I don't value his opinion as highly as active climate scientists. So, if there's a contradiction, I know where the safe bet lies (so to speak).

Al Gore

I don't get my science from Al Gore. That being said, his information is largely supported by climatologists.
http://www.realclimate.org/index.php/archives/2006/05/al-gores-movie/

He also doesn't seem to have much faith in the IPCC which you hold so dearly as the final answer on all that has to do with global warming.

The IPCC report collated the current understanding of climate science and related fields. It's certainly not the final answer of anything, but it's completely ludicrous to throw it out completely because you don't like its conclusions. By all means, criticize parts of it as necessary, but make sure you're on solid scientific footing instead of, say, Christopher Monckton.
http://www.realclimate.org/index.php/archives/2009/05/moncktons-deliber…

Coby,

Read your work and you know that it is purely unsupported conjecture. Your suggestion that a vulcanic event cause the cooling is baseless. The effect, from the studies, of vulcanic events ceases, show they cease to have a global impact after 7 years. There is no indication that we had major vulcanic activity every 7 years over multipule centuries.

Further you say that result of a decrease in solar irradiance dropped during that period but I cannot find any studies that show this. Can you point me to some? I can find studies that show that solar activity but not irradiance.

We know that there is an additional climate driver that causes warming and cooling. You say that MC are weak and once they start the warming CO2 takes over. This is not supported by the studies or historical fact. Toggweiler (2007) CO2 was responsible for less than 30 percent of Antarctic warming during the glacial terminations.

Further Hunt (2006) shows

While a number of characteristics of the MWP and the LIA could have been partially caused by natural processes within the climatic system, the inability of the model to reproduce the observed hemispheric mean temperature anomalies associated with these events indicates that external forcing must have been involved. Essentially the unforced climatic system is unable to sustain the generation of long-term climatic anomalies.
It stands to reason that whatever external forcing that caused the cooling had stopped and the temperature was moving back to the higher equilibrium in the absence of said forcing.

Clearly there is a climate driver that is not CO2, vulcanism, or solar irradiance. I have presented studies that show that what ended the LIA, or caused the LIA, was not CO2 and was not part of the natural process in the models. At this point you do not know how much if any warming has been due to CO2.

Now that all the hand waving has finished by Adam et all, can we please return to the topic. I first posted a article by Monkton about IPCC over exaggerations. This article was dismissed out of hand by Coby because he does not like Monkton. OK fine here is another article (not by Monkton) which casts serious doubt on the AGW theory.

Have a read,

http://climatesci.org/2009/05/05/have-changes-in-ocean-heat-falsified-t…

What the? Where is the outcry? Where is the personal attacks on the writer?

This article shows AGW theory is either wrong or seriously flawed.

The atmosphere is cooling, the oceans are cooling so where is the heat? Where is the global warming?

I have been accused by people on this site of a few things recently, namely a denier as opposed to a skeptic (how childish some people are) anyway i will now start calling some people "avoiders of fact" as any article that competes with the jones town cum Waco doctrine is first attacked if possible, if not ignored.

Good day to all you avoiders of fact

Round and round we go.

Have you seen the latest RSS and UAH temp anomolies? Looks like a downward trend to me. At the very least, they have leveled off. How can that be when Co2 keeps going up? I'm sure someone has a perfectly logical convoluted answer.

Adam, Forgot to add this. I suppose you also dismiss Dr. Roy Spencer as being unqualified or not as qualified.

Steve,

Looking at monthly is meaningless unless your comparing Aprils.

All April data from UAH:

09 0.09
08 0.15
07 0.24
06 0.18
05 0.41
04 0.20
03 0.20
02 0.30
01 0.25
00 0.11
99 0.05
98 0.65
97 -0.21
96 -0.05
95 0.21
94 -0.05
93 -0.22
92 0.18
91 0.13
90 0.01

The data set goes back to 79 if you want it. 79 was -0.17. 80 was 0.14.

What does all this tell you?

Steve,

This tells us what we already know, that there has been no warming since 2000. All I meant was that your statement was not concise and left a lot up to be misconstrued.

Steve -

I suppose you also dismiss Dr. Roy Spencer as being unqualified or not as qualified.

Dr. Spencer's degree is in meteorology. Also, he's a firm believer in Intelligent Design, which renders his opinion on anything else highly suspect.
http://www.uncommondescent.com/intelligent-design/roy-spencer-on-intell…

What does all this tell you?

It tells me, after I plotted the numbers you provide in excel, and plotted a (linear) trendline, that temperatures are increasing, with lots of variability (weather) superimposed over it. If you'd bothered to do the same, you'd have seen this, and I wouldn't have to put up with your inane nonsense.

This obviously is not the best way to discern a long-term trend, but for that, you should check out the IPCC 4th assessment report.

Adam,

You don't have to put up with anything. No one is holding a gun to your head to respond. Are they? Sounds like someone is getting a little frustrated.

"temperatures are increasing, with lots of variability (weather) superimposed over it." That's not a very concise statement and leaves a lot to be misconstrued. To quote Vernon. Temperature increases are no where near the projections by your beloved IPCC. They are not even increasing anymore. At least Vernon can admit that.

All apologize for the inane nonsense.

Steve,

I was merely pointing out where you were not being precise because people like Adam will twist your statement into something stupid.

Yes, world sea ice is approaching the 1979-2001 mean. Yes, we have not been warming since 2001. Yes we know that tree-ring do not work well as temperature proxies:
DâArrigo et al (2006)

There are several hypotheses for this divergence [Jacoby and DâArrigo, 1995; Briffa et al., 1998; Vaganov et al., 1999; Barber et al., 2000; Wilson and Luckman, 2003; DâArrigo et al., 2004; Wilmking et al., 2005], none of which appear consistent for all NH sites. Although we calibrated to the common 1856â1978 period, valid calibration using a reduced data set would be possible until the mid-1980s (Figure 5). After this period, however, the divergence between the tree-ring and instrumental data results in weakening of calibration results and failed verification statistics.

I know that Fedorov et al (2206) showed that 5 to 3 million years ago, globally averaged temperatures were substantially higher than they are today, even though the external factors that determine climate were essentially the same. That is the CO2 level is the same as it is today. CO2 does not explain that warming.

I even know that warming in the Antarctic started while the MC was causing a reduction in solar irradiance in the SH.

I know that there is no point where CO2 has been a significant climate forcing.

Steve, Vernon,

I think you guys don`t understand what it means to determine a trend out of noisy data. I am also not at all sure what you think it proves to look at every April over the last 30 years. What does that prove? What are the trends in other months? Why not use all the data, instead of just one twelfth?

Regardless, you guys might want to check out this article:
http://scienceblogs.com/illconsidered/2008/09/signal_vs_noise.php

Coby,

I really think you should reconsider that statement. It shows that you do not understand trend analysis. Doing a trend of Aprilâs temperatures over the past x number of years is completely proper. It is a valid way to show the trend temperature trend.

Once again I will point out that your article does not address this issue at all. Your article is about the length of time to determine a trend.

Coby,

What does anything prove? Nothing. Pick whatever timeframe you want. It doesn't matter. How long do temps have to go down before you will consider it a trend and not noise?

How on the one hand can CO2 be THE major factor in global warming, but on the other hand, so-called noise seems to interupt or delay the warming? What are the factors that are currently creating this noise?

You want your cake and eat it too.

Rather than squabble over a small and somewhat questionable increase or decrease in temps (did you read my link)

The avoiders of fact have once again successfully avoided the article which states using facts that the oceans are cooling now for over 6 years.

The AGW theory clearly states that as you increase CO2 you trap/snare/restrain etc the IR leaving the Earth, this in the end leads to an increase in global temps. Which is what you lot are arguing about now.

However the theory does not end there, no this atmospheric heat has to go somewhere, once again according to the theory the oceans temps will increase as it absorbs all this extra heat.

As the latest emphircal data (not models) shows us the oceans are cooling, so where is the extra heat?

How can the AGW theory be correct when the oceans and upper atmosphere (re hotspot) are not warming?

Coby your silence on this is deafening are you in denial?

Feel free to debate me on this issue rather than just show your ignorance of the science at hand.

Cheers

Crakar

Well,
I guess Crakar successfully sums up the facts to date in post 29 and 83. As for the theories, we'll just have to wait and see. It's a good thing we don't have spend dramatically now to alter something we're not sure about.

And, I guess I'm most pleased the polar bears will survive.

By Paul in MI (not verified) on 09 May 2009 #permalink

Now hereis an article which covers a whole range of issues so i thought itbest to put in here.

Now i know some people will avoid the VERY IMPORTANT FACT that all the IPCC models are completely wrong in an upward direction and will avoid the fact that the planet is cooling in every measurable way dispite the fact that CO2 IS INCREASING by cunningly trying to belittle the website and/or the author.

But what you need to realise here is that no matter what they say the facts are still the facts and no religious buffoonery will change that.

Enjoy reading this article

http://scienceandpublicpolicy.org/images/stories/papers/originals/april…

"There is no "proof" in science" - wrong! The "proof" of a scientific theory or hypothesis is that it should explain past data and predict future phenomena. The AGW hypothesis has done neither.

[This is not proof, this is being consistent with. There are often many ways to explain available data and just because a given theory is consistent with it does not prove it is correct. It doesn't even "prove" it. You are trying too hard just to miss the point. AGW is consistent with all the major observational and experimental data.

-coby]

"In the case of the theory of Anthropogenic Global Warming, what we do have is a theory (first conceived over 100 years ago) that is based on well established laws of physics, it is consistent with mountains of observation and data, both contemporary and historical, and it is supported by very sophisticated and refined Global Climate Models that can successfully reproduce the climate's behaviour over the last century." - WHAT A LOAD OF UNDILUTED UNMITIGATED HOGWASH!

[Try bold as well as all caps, it will make you even more correct!
- coby]

1. The "AGW theory" was not conceived over a 100 years ago. What Svante Arrhenius published in 1896 was a paper in which he proposed the hypothesis that, based on Stefan Boltzmann's law, "if the quantity of carbonic acid increases in geometric progression, the augmentation of the temperature will increase nearly in arithmetic progression". Based on this hypothesis he predicted that if CO2 in the atmosphere doubled the Global temperature would increase by 5-6 degrees C.

The "AGW Theory" or Hypothesis states 2 things both vital to each other 1. Global Temperatures will rise due to increasing levels of CO2 in the atmosphere and 2. This rise of temperature and rise in atmospheric CO2 will be catastrophic to our planet.

[I don't think this is the common interpretation of AGW. The consequences of warming are far less certain than the simple fact that human emissions of CO2 cause the global temperature to rise (which is the basic principle of AGW, as first proposed by Svante Arrhenius
- coby]

Svante Arrhenius on the other hand made it quite clear in his subsequent book published in 1906, that he felt that anthropogenic CO2 would be needed to offset another ice-age and that rising global temperatures were needed to feed a growing world population! He thought anthropogenic CO2 would increase global temperatures but the effect would be entirely beneficial. It so happens he was wrong in his first prediction that CO2 would raise global temperatures significantly but right in assuming that a warming Earth, for whatever reason, would be beneficial to mankind.

[He also thought it would take humankind 3000 years to double CO2 levels, a fact which is extremely relevant to one's views on the balance of effects on the planet
- coby]

"..the theory of Anthropogenic Global Warming, ..is based on well established laws of physics" ... HOGWASH! You cannot, as Svante Arrhenius did, base a hypothesis on one assumption when there a myriad of other factors at play, then calculate temperatures based on that assumption alone according to the laws of physics, and then claim that the entire hypothesis is founded on the laws of physics, thereby implying it has the same authority as the original laws!

[Here are 15 factors discussed in the IPCC reports and commonly integrated into GCM's: http://www.grida.no/publications/other/ipcc_tar/?src=/climate/ipcc_tar/…
This is by far not the entirety of the equations and physical principals involved. Your assertion that GCM's are based on "one assumption" merely reveals your near complete ignorance of the actual science, which is remarkable given the apparent strength of your convictions!
- coby]

The proof of the pudding is in the predictions. Svante Arrhenius predicted that Global temperatures would rise by 5 to 6 C if CO2 was doubled. The Charney report predicted that global temperatures would rise by 1.5 to 4.5 C. The figures from NOAA - from 1880 to 2007 CO2 rose from around 280 ppm to around 387 ppm an increase of 38.21%. In the same period globally averaged land temperatures increased by 0.64 C and sea surface temperatures by 0.12 C. We can thus compute Globally averaged temperatures to have increased by 0.276 C from the annual mean. Were CO2 to increase by 100% by extrapolating (assuming again if temperatures were to continue to increase) temperatures would rise by 0.72 C. Charney report fails Arrhenius fails. The hypothesis is wrong. .. to be continued

[You make two basic mistakes here. Firstly, CO2 forcing is not the only factor controling climate, and second is the time delay between the imposition of a radiative imbalance and the arrival at a new equilibrium. Due to the large thermal inertia of the oceans, it would take several decades after the cessation of CO2 rise for the earth to stop warming from this factor. I have an article about that here.
- coby]

Coby, your objections dont hold water. I will reply soon when I have time. By the time I finish hopefully you and your readers will recognise the AGW hypothesis for what it is - wrong and totally unsubstantiated, or in other words, respectfully, hogwash

"AGW is consistent with all the major observational and experimental data" No it is not consistent with "all the major observational ..data" I dont know what you mean by "experimental data". So far as I know no experiments have been carried out with the Earth's climate. If you mean consistent with the experimental models - these are irrelevant to observational data, which will prove or disprove the hypothesis.

A scientific theory or hypothesis has to explain existing facts and data as well as make accurate predictions of the future to be valid. For example for the special theory of relativity to have any hope of acceptance it must explain all the equations of Newton, planetary motions etc which explain the real world very nicely already, thank you very much, and more. It does.

The ultimate AOGCM of the IPCC is not consistent with observational data past. How do we know? The IPCC tells us so.

Just look at the global surface temperature graph from 1900 till 2001. It looks nothing like the temperature graph computed from actual data by NOAA. After 2001 the temperatures really take off. How much depending on various scenarios and models, but all take off at an accelerating rate. And we are supposed to believe this, based on the model which fails to replicate the past.

There are many other deviations. Just a few admitted by the IPCC 2007 report- Antarctic sea ice extent shows no statistically significant trend, a lack of rise in the surface temperatures across Antarctica, no trend in tornadoes, annual numbers of tropical cyclones, hail, lightning and dust storms (Al Gore â inconvenient truth?). No overturning of the meridional global ocean circulation. (Gulf stream stopping or the Humboldt current reversing etc.)

No longer are the deviations between the models' computed climate characteristics and the observed data referred to as errors they are now referred to as biases. Facts have a bias the truth lies in the models (forgive the pun).

What about the future? Well based on this same AOGCM that fails to model the past the IPCC reaches the conclusions that it is âvery likelyâ that global warming is caused by "human activity". There are a host of âVery Likelyâ and âLikelyâ predictions the IPCC has made but a quantum leap from the science expounded in the main body, which is cautious, to the conclusions which give no explanation of how they have been reached.

The Summary of the IPCC 2001 report says that the confidence of the projected climate change of Higher maximum temperatures and more hot days over nearly all land areas for the 21st century is "Very likely". However the observation of such an occurrence of such a phenomenon in the climate of the latter half of the 20th century is only "Likely". Hmmm.. events that have already happened have less confidence than what is about to happen in the future, based on these wonderful models!

The IPCC is aware that poor modelling of existing data is not a good look for its models. It reveals that some of these models include âflux adjustmentsâ. Another name for âflux adjustmentsâ and other factors with no explanations â âfudge factorsâ, arbitrary multiplications, divisions constants added to make models fit data.

You state at one stage âYour assertion that GCM's are based on "one assumption" merely reveals your near complete ignorance of the actual scienceâ â I did not state that the IPCCâs âGCMâsâ or AOGCMs are based on one assumption, though I was coming to that, Arrheniusâ model was. But whether you model one assumption or 15, the basic principle remains the same â if you do not take all factors affecting climate adequately into account and you do extremely scientific and sophisticated computations with the rest of your assumptions, there is no guarantee or indeed any confidence whatsoever that your model will relate to actual climatic reality.

It is ironic you mention ignorance while directing me to the IPCC website on the scientific basis of their models. Of the 15 factors discussed in their âradiative forcingsâ graph, there are such things as âSolar Irradianceâ, âStratospheric water vapourâ, âLinear Contrailsâ which includes such things as âcloudinessâ and âCirrusâ. Now these you will admit are not minor influences on the climate and on global temperatures. Yet the IPCC admits for all these the level of scientific understanding is âvery lowâ.

Low scientific understanding translates to ignorance by the IPCC and indeed, in all fairness, by the scientific community, of these influences. The level of their scientific understanding for CO2, CH4, NO2 and Halocarbons is however âhighâ, (no wonder with all the billions of dollars spent on lopsided research on these factors) and, amazingly enough, on their graph, the âradiative forcingsâ of these items far outweighs those of all the other factors taken together, for which the level of scientific understanding is for the most part âvery lowâ.

Now it is well known the greenhouse effect of water vapour and clouds far outweighs that of CO2. So in order for the AGW hypothesis to be valid, (that indeed CO2 is the current driver of global temperatures), there has to be some multiplication effect of this CO2 by increasing water vapour and also somehow a positive effect on global warming by the clouds. But in all the huge research put into global warming CO2 research there are still no figures of the average water vapour content of the atmosphere or indeed any graph of the variation of water vapour content over time, as we have for CO2. The same applies for clouds. All we have is one article in a scientific journal suggesting that this might be so, with no data or empirical evidence to back it up.

No evidence for this basic assumption? ⦠Hmmmm⦠to be continued

If i may add to this thread, in regards to the IPCC's "level of scientific understanding" graph, i notice o3 is considered to have a slight cooling radiative forcing.

This i find strange as o3 would trap UV entering and leaving the atmosphere (causing it to warm) also o3 coupled with o2 would combine to have a significant effect on the higher atmospheric temps.

Am i right in this assumption or have i missed something? Added to that as the Sun has been dormant the last few years the amount of UV being recieved on Earth has reduced in many orders of magnitude thus reducing the warming.

Just a thought.

Also the IPCC seem to have left off another element that contributes to the warming, and that is water vapour. Is there any logical reason for this.

Cheers

Crakar

Crakar I do not know whether you are right on whether O3 would be cooling or warming. I am not an expert on this matter, but if I was a betting man I would lay my money on IPCC being right on this and you being wrong. I just dont want to get sidetracked into issues that are not relevant to the main issue which is that the AGW hypothesis is totally unsubstantiated.

I have broadly laid out the case in which I have touched upon the weakness of the mathematical models, the failure of these models to replicate past data and about water vapour and clouds, but these need to be elaborated as they are pretty crucial to the issue. I shall do so subsequently again when I have time.

In the meantime I want to touch upon a related issue of the AGW hypothesis, which is that the global temperature rise will be catastrophic to our planet. Part of the reason is that it predicts a catastrophic rise in temperatures within a very short time, which could inundate large populations.

Coby writes "I don't think this is the common interpretation of AGW. The consequences of warming are far less certain than the simple fact that human emissions of CO2 cause the global temperature to rise (which is the basic principle of AGW, as first proposed by Svante Arrhenius". I dont know what on earth he is talking about. If the consequences of the alleged AGW were not very bad to catastrophic it would be put in the dustbin as Arrhenius' theory was. Who would bother? But dire warnings are rising to a cresendo. The precautionary principle is being invoked. Action now before it is too late. It has become the defining ideology of our generation. No more are we impotent observers of climatic change, we are saving the world.

It is therefore important to address the basic evidence. Are global temperatures rising? (I can see hackles rising over this - yes they have risen a bit over the last few decades - but the questions are will they continue to rise? Is this rise unprecedented? Is it due to CO2? These questions need to be addressed)

All this for a later time... to be continued...

My apologies Cracker - I also cannot understand why O3 would have a cooling effect. Doubtless they have an explanation. It is puzzling but not crucial to the issue.

Are global temperatures rising? Is the (alleged) rise unprecedented? Is it due to CO2?

Prof Bob Carter of Australia rightly points out âthe public debateâ (on the Global Warming hypothesis) âis bedeviled by the lack of context and lack of balanceâ. He then goes onto ask the question â Is it warming or not? And he quite rightly points out that depends ...

Since the last Ice age 16,000 years ago if we plot the trendline of the temperature graph till date, it goes up. So we can say the Earth has been warming since the last ice age. But 10,000 years ago we entered a period called the Holocene, where the temperatures had warmed approximately to what they are today. If you take the temperature graph from the beginning of the Holocene to date and fit a trendline to the curve we find it goes downwards slightly so the temperature has been cooling since the last 10,000 years.

What about the last 2,000 years, since the time of Christ? Well the trendline from that period also goes downwards, but goes downwards at a faster rate. So the cooling increased since then. The temperature graph since the little ice age the last 700 years? The trendline is horizontal - no change since 700 years ago. What about from 1972 to 2007 a period of 35 years. It goes up like a rocket (plotted from NOAA data). The trendline shows a warming of 0.18 C per decade. This is the supposed empirical âproofâ of GW and specifically of AGW.

But what about 1998 to 2006? The trendline is horizontal - no change. 1998 to 2008 a slight cooling.

In view of this David R. Easterling of NOAA and Michael Wehner jumped in to say that this cooling is statistically insignificant. Of course it is. But is a period of 26 years from 1972 to 1998 statistically significant? OF COURSE NOT! It is too short a period over the data stats spanning the Holocene period. Are there any other periods of 26 years or so, in the Holocene period, when the temperature has climbed AT THE SAME RATE only to come down again later? Yes many.

And there is another thing â In IPCCâs reports there was no room for temperature to be static or even declining with rising CO2. Even if CO2 leveled off the temperatures were set to rise. But CO2 has increased by 5% over the 10 years when temperatures havenât risen.

Well this was finally noticed by the modelers, who brought out a paper in 2007 to say they hadnât accounted âinternally generated natural variabilityâ in their models and hence a failure to predict the temperatures on a decadal level. They couldnât possibly have predicted Pinabuto, other volcanic eruptions, el Niño etc. But now they can and incorporated this into a new model and lo and behold over the next decade internal variability in the climate system will initially offset the effects of anthropogenic warming.

But then - whamo! 2014 will be 0.3 C warmer than 2004 and from 2009 to 2019 at least half the years will be warmer than 1998 â the warmest year on record.

Well 2009 is this year lets wait and see.

Ok so global temperatures have risen from 1972 to 1998 and leveled off since then. Thems the facts. But is it due to CO2? ...

oops this was posted in the wrong place

From ice-core records we can see a high correlation between GTG (greenhouse trace gas) concentrations and temperature variations over 420,000 years and through four glacial cycles. In 1999 a paper was published showing that CO2 increased several centuries after the warming of the last three deglaciations.

Something caused the Earth to warm and come out of the ice ages â it was not CO2 or GTGâs. But why did CO2 go up later? Well one explanation that seems reasonable to me is the change in land ice coverage and the buildup of the terrestrial biosphere. As the ice sheets retreated grasses and trees moved in CO2 and methane built up, further increasing the temperatures due to the greenhouse effect.

Then something caused the temperatures to fall and the build up of CO2 and other GTGâs were not enough to offset this. CO2 levels remained high as the temperatures plunged sending the Earth into another ice-age.

What was that something? The IPCC and coby have been at pains to tell us that the TSI of the sun varies only slightly â it couldnât possibly explain the changes in global temperatures. ITS NOT THE SUN STUPID!

To me its clear that CO2 was not the driver of the climate in the past. It was the sun â DIRECTLY and INDIRECTLY. But that is for another time.

The dominant climatic feature of the Earths climate in the last 2 million years have been the ice ages, interspersed with relatively brief warm periods of about 10,000 years. We are currently in an interglacial period. The longest on record. What we really need to fear is another ice age not warmer weather.

Svante Arrhenius was a genius. Maybe he was right - anthropogenic greenhouse gasses may stave off another ice-age for another few centuries.

Coby writes "Correlation is not proof of causation. There is simply no proof that CO2 is the cause of the current warming" and then goes on to argue against this statement.

By writing that âCorrelation is not proof of causationâ Coby is falsely implying that there is a strong correlation between CO2 and Global temperatures whereas actually there is an extremely poor correlation between them. And in normal circumstances the hypothesis would be rejected.

When you have two independent data sets, say global temperatures and CO2 levels, statistical correlation, whereas not a proof, actually strongly indicates whether two variables influence each other or they donât. If the correlation is less than 0.3 the chances are that they have no influence on each other.

Someone said that you are entitled to your own opinions but not to your own facts.

When it was pointed out by Tim Wells right at the beginning of this blog that actually there is no correlation Coby directs him to a site where is says âNone of the advocates of the theory of Anthropogenic Global Warming claim that CO2 is the only factor controlling temperature in the ocean-atmosphere climate system. ... AGW only makes the claim that CO2 is the primary driver of the current warming trend, as in the rapid rising seen over the last 100 years. ...â

What the devil has this meaningless drivel, which has its own false statements welded into it, got to do with the fact that there is in fact very poor correlation between the temperature data and the CO2 rise data over the 150 years that we do have direct global temperature data and ZERO correlation over the last decade?

(There is no ârapid risingâ of temperatures âseen over the last 100 years. ..â and poor or no correlation between temperature data and CO2 data indicates that CO2 has very little or nothing to do with global temperatures)

What has that got to do with Coby FALSLY implying correlation when there is none.

From February 1998 to February 2008 Global Temperatures data from BOTH Satellite (UAH MSU lower troposphere) AND Land And Ocean Variance Adjusted Surface Temperatures (Hadley CRU T3v) when plotted and correlated against Scripps monthly CO2 data from Mauna Loa â the correlation (r squared values) between Temperatures and CO2 values for both turns out to be ZERO, ZILCH, 0

Now the last decade it maybe argued is not a fair test for correlation, but the IPCC and the AGW gang have precisely been arguing that the ocean and other factors have been masking CO2 driven Global Warming, but now we have entered the really dangerous phase. So surely a correlation should be even stronger now than before and not drop to zero.

And for 1880 to 2008 the correlation is very poor and gets even less as the time interval is increased.

Here is a graph that shows absorption spectra of green house gases

http://www.iitap.iastate.edu/gccourse/forcing/images/image7.gif

As you can see o2 and o3 trap/absorb all of the UV spectrum and some of the IR spectrum as well, so yes they would play a part in cooling wouldn't they? I think not. And as i said earlier where is the water vapour?

Having said that i suspect Coby's comeback would be these figures are relative to 1750 not the actual figures of the green house effect.

Which leads me to water vapour again, can anyone tell how much water vapour is produced from a nuclear power plant, or coal fired or gas fired plant? Or in a Hydro electric plant? I suspect the answer is a lot more than CO2. What about all that irrigation in the deserts we like to do now or how about all that extra water we are pumping out of the artesian basins around the world.

You see water vapour is by far the most powerful GHG as shown by the above graph and yet the IPCC never mentions it, oh well they do because without it the AGW theory is dead in the water (pardon the pun).

So in summary the IPCC claim that CO2 is the driving force of AGW but can only be with the help of water vapour, but water vapour cannot be the driving force of AGW because it does not last long enough in the atmosphere (hydrologic cycle) http://www.fwee.org/hlogic.html

So it seems you can have your cake and eat it too.

Just some points to ponder as we wonder what causes the warming (or cooling of recent years).

>The warming rate from 1975-1998, when humankind might have had some influence, was the same as the rate from 1860-1880 and from 1910-1940, when humankindâs influence was negligible.

>For 15 years there has been no statistically-significant âglobal warmingâ.

>For 8 years there has been global cooling.

>The greatest warming rate in the past 300 years was from 1645 to 1715, before the Industrial Revolution began. That warming rate was eight times the 20th-century warming rate.

>The warming of the past 300 years is indeed unequivocal, but the mere fact of the warming tells us nothing of its cause. There is no scientific basis for attributing most of it to humankind.

>The largest-ever survey of scientific opinion â the largest to date â found more than 31,000 scientists did not consider the human contribution to âglobal warmingâ significant enough to be dangerous.

By Paul in MI (not verified) on 30 May 2009 #permalink

Paul,

Point one and four: can we have some numbers and sources for these claims please?

Point two and three: these are very short time periods for determining a climate change signal. For your 15 year claim, I suspect you are looking at the error bars and seeing an overlap. Given the relatively small trend that is expected compared to interannual variability and the ~.1oC+/- error range I think we must expect that it takes more than 15 years to meet your definition of statistically significant.

Point 5: "no scientific basis"...sure. That is why we call you denialists, you deny the existence of copious volumes of long standing and new research and just say "no evidence". Plus I am unaware of any global temperature record that indicates a 300 year long warming period, care to provide a source?

Point 6: the oregon petition is not a proper survey of anything, much less of climate scientists.

Coby,
Thanks for your response, but I really don't want to debate the exact status of the numbers or how long a data set is.

The basic graphs are bandied about and discussed here and elsewhere. For every bit of data supporting this point or that point, someone else points out a bit of data which counters it. You can see it for yourself and of recent times, live it for yourself. These are points to ponder, no more, no less.

Whether AGW is real or not will only be solved by witnessing what happens in the future. I don't hope to prove it or disprove it here.

By Paul in MI (not verified) on 03 Jun 2009 #permalink

Coby (and Adam),
Assuming your analysis is completely correct and we need to do something:

What is it you want the US (or world) to do?
What would it cost?
And what would it accomplish?

Isn't that what needs to be put in front of the average citizen? And doesn't that become more difficult if temperatures continue to drop year after year?

By Paul in MI (not verified) on 03 Jun 2009 #permalink

Paul in MI -

Assuming your analysis is completely correct and we need to do something:

These are fair questions.

What is it you want the US (or world) to do?

A good launching point would be with the Princeton stabilization wedges; implementing policies to support these kinds of initiatives. Cap-and-trade is a good start, but wholly insufficient. I'm still undecided whether a positive incentive (e.g. subsidizing cleaner technologies) or a negative incentive (e.g. penalizing dirty technologies) is better. I probably tend towards the negative incentives, since then the free market will naturally eliminate the "bad" alternative technologies, whereas subsidies can lock us into supporting a "bad" technology (like the corn ethanol debacle).

What would it cost?

Probably a lot less than people think.
http://web.mit.edu/globalchange/www/MITJPSPGC_Rpt146.pdf

And what would it accomplish?

Ideally, stabilizing CO2 concentrations and limiting the inevitable temperature increase from something disastrous to something manageable.

Adam,
thanks for responding in a straight forward manner.

Now let's dial it in a bit>

While I agree on the ethanol issue, I am not for subsidies, positive or negative. But let's leave that alone for now.

I think by wanting cleaner technology, you mean lower CO2?
If correct, how much lower would you want to go? Say as a percentage of total emissions today?

And if cost is hard to determine, ok, I agree. So how much would you say is ok to spend per individual, per year, to accomplish this? $100? $1000? $10,000?

Then with this reduced CO2 output we limit the inevitable temp increase by ?? how many degrees? And is it worth it?

By Paul in MI (not verified) on 03 Jun 2009 #permalink

re #97, #97, #99 - Paul,

I am using this data, which is consistent with all the other intrumental records I am aware of.

The trend from 1975 to 2000 was .2oC/decade. 1860-1880 was less than .1oC/decade. 1910 to 1940 was .17oC/decade. I know it is easier to make stuff up and then decline to substantiate it when asked, but this is also dishonest. I asked you for sources and numbers because I knew you had none or they did not support your claim.

Your claim of 8x's faster warming 300 years ago is ludicrous. Show us the evidence or stop polluting the information age with crap.

Paul in MI -

I think by wanting cleaner technology, you mean lower CO2?

Mostly, Yeah, but more generally I mean greenhouse gases (in the context of this conversation anyway). I realize that (strictly speaking) it's not the right term, but it's convenient.

If correct, how much lower would you want to go? Say as a percentage of total emissions today?

This is a question where there's a huge amount of debate, but the number I generally assume is around 80% reduction by 2050 relative to 1990 levels (CO2 equivalent). This is more of a guideline, though, and I'm not particularly committed to it over other targets. The point, really, is that cuts need to be pretty drastic by mid-century.
Here's a reasonable start, where I base this target on:
http://www.ucsusa.org/assets/documents/global_warming/emissions-target-…

And if cost is hard to determine, ok, I agree. So how much would you say is ok to spend per individual, per year, to accomplish this? $100? $1000? $10,000?

In short, yes. Though, I don't think the high end of your range is a reasonable amount. Every single study I've seen gives a much lower annual amount per family. There's a reason that Republicans had to misrepresent the study I cited above to show a high annual cost. If there were legitimate studies that showed high annual costs, you can bet your bottom dollar they would have been all over it like white on rice.

Then with this reduced CO2 output we limit the inevitable temp increase by ?? how many degrees? And is it worth it?

If we can stabilize atmospheric CO2 at 450ppm (and an appropriate CO2-equivalent level for other greenhouse gases, though I can't give you a specific number), temperatures are likely to increase only 2 degrees or so above pre-industrial levels. And if the alternative is 6-7 degrees (as more recent studies seem to suggest) under a business as usual scenario, well, yes, it's totally worth it. Even if business as usual results in a more conservative estimate of 4-5, that would still have very unfortunate consequences (to put it lightly)

Coby,

Point 1, As for the other years, I don't disagree with your figures. For short term (30 yrs, or weather as you call it) it shows similar rises in temp, not induced by CO2 rise.

Points 2 and 3, we agree then. Though I agree the time frames are short and at this point statistically insignificant, so we shall see.

Point 4, 8x figure from 300 years ago was misleading. This was a "regional" value. Not really useful here. Sorry for that.

Point 5, We'll have to disagree on this until the models do a better job of predicting the future.

Point 6, We'll have to disagree here too. If you want to call 'em "regular folks" instead of scientists, that's ok with me, but I'd like to see more of 'em being convinced.

By Paul in MI (not verified) on 03 Jun 2009 #permalink

I have yet to see a list of Al Gore's scientific qualifications. Can anyone provide them?
Also, how many scientists stand to gain in some way by the nonsense being disseminated by the UN committee on climate change, that carefully chosen group of persons. Monetary gain? Increase in personal status? More government funding?
As a government medical scientist myself, I notice that dissenters to the climate hoax are now afraid to speak out for fear of execution by people like our (Dr?)Coby.
There seem to be similarities to the Catholic Inquisition wherein over a 350 year period, dissenters numbering around 65 millions were tortured to death by the most ingenious means, and they got away with it too....

By Dr.K.Hall (not verified) on 16 Oct 2009 #permalink

Good grief, "Dr". K. Hall, what a load of nonsense cramped in so few sentences.

You "notice" as a "government medical scientist" that "dissenters" are now afraid to speak out "for fear of execution". Really. Is that why Roy Spencer has a blog? Roger Pielke Sr.? Steve McIntyre? Is that why Fred Singer and his organisation keep on throwing out reports and meetings containing very shoddy science? Don't you see how extremely wrong your claim is? Most likely you do not. You are so afraid that AGW is correct, that you put your fingers in your ears and say "la, la, la, I can't heeeeaaar yooouuu".

Methinks Dr K Hall has been self medicating. Naughty, naughty.

By Dappledwater (not verified) on 16 Oct 2009 #permalink

There seem to be similarities to the Catholic Inquisition

Hey, Marco, bring your stake; DW, get out your rope; and I'll bring my matches just like the last time someone claimed that AGW is false!

There are FOUR examples of exactly what Dr Hall is saying in a row!!
Insult, ridicule sarcasm and diversion.
How do you people justify your writings??
Do you really think this way in real life?
Are you actually adults??
Ooooh! It makes me so mad!
To clarify the basic comprehension of the english language for you guys, Dr Hall's reference to the Inquisition is a METAPHOR for the attitude of believers towards heretics, skeptics, non-believers, faithless heathens, etc. Note the abundance of "religious" type words.
Very appropriate in reference to the ZEALOTRY of the AGW movement.
I look forward to your replys to me. What gastly means of abuse and torture can you come up with, I wonder?
I wonder which sentences, or phrases or words will appear in said replys in inverted commas for special mention? Perhaps, like my last few posts, I won't get a reply at all.
The anticip..... pation is exciting anyway.

Michael, the term you are looking for is honesty, something which deniers show a complete lack of.

When a denier says something wrong the first time it can be put down to ignorance. However, when it is repeated over and over again and is continually being shown to be wrong then that changes it from ignorance to dishonesty.

Hall is dishonest (if he is what he says he is) since he should know better. I have a similar background and my studies included basic chemistry and physics which enabled me to understand climate science.

By Ian Forrester (not verified) on 17 Oct 2009 #permalink

On the contrary Ian Forrester.
I would like openness and honesty from you!
In what actual way is Hall being dishonest?
Which sentence of his is "dishonest?
What reason would he/she have to BE dishonest?
What justification is there for "deniers" to be dishonest?
I am including a link to a non-climate but equally political topic: Poverty.
I ask all readers here to point out to me exactly what the "action" is.
Are they giving out money? Are they creating industry, jobs, or work ethics? What is the actual point of this "Standing Up"? If Australia's 20 Million people "Stood Up" what is the Prime Minister to do?
Personally, I don't like poverty at all. I wish there were no such thing.
http://www.standupagainstpoverty.com.au/
My point is that the entire AGW movement is about "feel good" religious zealotry!

I would appreciate a reasoned response.
(I do appreciate that your reply was not "Inquisitional")

Michael, do want us to seriously engage in a discussion with someone who only came here to scream "CONSPIRACY!" ?
THAT's what is dishonest in the response of 'Dr. K. Hall'.

Note also that he put the "Dr" in his signature there on purpose ("look how smart I am"), since he started with a jibe at Al Gore, and tried another one at Coby ("Dr?"). I have yet to see anyone with this arrogange engage in a meaningful discussion.

Regarding reasons for deniers to be dishonest, there are many. The most common reason, however, is that they are afraid any action to mitigate climate change will cost them money, and/or requires them to make changes in their life, changes they don't have full control over.

"There are FOUR examples of exactly what Dr Hall is saying in a row!! Insult, ridicule sarcasm and diversion................Are you actually adults??" - Michael

Yeah, but HE started it!. Insulting the divine spiritual leader Al Gore.!. I just don't understand why deniers say AGW is a religion.

By Dappledwater (not verified) on 17 Oct 2009 #permalink

"I notice that dissenters to the climate hoax are now afraid to speak out for fear of execution by people like our (Dr?)Coby."

Where's the METAPHOR!!! in this sentence?

The person authoring that sentence is a kook, and deserves the kook-treatment being dealt out.

Richard,

Thank you for your reasoned and methodical series of comprehensive posts. I've found them very helpful. I remain mystified as to why there was so little similarly reasoned response to them, especially from Coby.

Wombat,

I replied extensively to Richard inline in comment 85. His subsequent posts were simply too long and similarily confused on very basic issues for me to address them or even read carefully. I look at blog threads as a conversation, if Richard wants to lecture then I will not likely attend!

What specific point of his are you interested in pursuing. If there are many, please pick just the one or two "best", and I will try to address them.

First let me say one thing very clear i am not posting this to debate the article but to ask whether the log graph of CO2 is accurate.

I do not care what archibald says OK.

Though i still expect to get "that guy belongs on the list with Monkton" So just to confirm please reply to the log graphs only and not what archibald says OK.

Remember comment only on the graphs, i dont care where he got 10% from just comment on the accuracy of the log graph OK.

http://wattsupwiththat.com/2010/03/08/the-logarithmic-effect-of-carbon-…

Crakar, the use of a log scale to show effects of CO2 is legitimate. The problem with the graphs shown on wattsuphisbutt is that the author uses the MODTRAN programme. This shows the forcing, or temperature change, associated with varying levels of greenhouse gases. You plug in any concentration of the various gases and it gives you temperature rise.

What it does not do is incorporate the various feed backs which are known to occur, the most important of which is water vapour feed back.

Thus the programme gives very low sensitivities since it do not include either short term feed backs or the long term feedbacks (change in albedo, GHG's from melting permafrost etc.)

By Ian Forrester (not verified) on 11 Mar 2010 #permalink

Thanks for the response Ian and sorry for my slow reply.

Now i do not want to turn this into a lowbrow slugfest as some would imply but if we agree on two things:

1, the graph is correct (minus feed backs)

2, the theory of AGW states the increase in atmospheric heat (see graph) will cause a much larger feed back from VW.

Then the graph shows if we increase the CO2 level from say 385ppm to 770ppm there will be very little increase in heat so how could this possibly cause a much greater increase in WV so much so as to bring about the temp increases predicted by the IPCC?

Any thoughts

I see we have successfully ignored the log relation ship of CO2, lets try this approach.

Coby states in the first para of his answer (see preamble)

"There is no "proof" in science, that is a property of mathematics. In science, one must look at the balance of evidence and formulate theories that can explain this evidence. Where possible scientists make predictions and design experiments to confirm, modify or contradict their theories and must modify these theories as new information comes in."

So lets look at the balance of evidence. The following link will address the scary stories promoted by those who stand to gain the most in this scam.

http://scienceandpublicpolicy.org/images/stories/papers/reprint/Congeni…

The balance of evidence suggests the theory of AGW is a crock of shit, anyone care to dispute this or will you all go very quiet as per the log graph i produced a couple of posts ago?

Crakar:
After establishing the logarithmic, *independent* relationship of CO2 and temperature with Ian, you asked,

Then the graph shows if we increase the CO2 level from say 385ppm to 770ppm there will be very little increase in heat so how could this possibly cause a much greater increase in WV so much so as to bring about the temp increases predicted by the IPCC?

Because the *little* increase in heat refers to the *independent* impact of CO2.

This is the key thing you arenât grasping. The increase in CO2 to 700 ppm does not occur immediately, with the limited impact on temperature (and thus water vapor) that an instantaneous increase would engender. It occurs, in future climate models, *along with all the other known positive feedback mechanisms, including water vapor, cumulatively, and over time*. (On a side note, the 6 degrees C upper bound that Archebald refers to is an artifact of the varying uncertainties associated with projecting the exact impacts of these feedbacks. The IPCC's central projection was about 2.0 deg C but I might be off on that. I forget.)

It was also interesting that your zeal at the revelation that CO2 has (by itself) a logged relationship with temperature is echoed by the ill-informed choir at Wattsup.

Observe some of their gloating posts, now that they, like you, think that the logged relationship of CO2 to temp is the latest silver bullet against AGW:

hey dude CO2âs contribution is logartithmic not linear â heck that scientific enuff for ya. . . . [unbelievable sophomoric arrogance]

The idea that water vapour will cause accelerated warming seems to rest on the idea that the earth is a consistent temperature . . . [which is an absurd straw man]

Iâm told the science is settled, but something as basic as this seems to cut right to the heart of the whole argumentâ¦how could something as simple as this have been overlooked? [And thatâs the point; it *wasnât*.]

Obviously thinking youâd scored a really devastating point, Crakar, and emboldened by what you perceived to be the silence of refutation, you later linked to a story about someone who will benefit from âscary storiesââwhich of course has absolutely *no bearing* on the question of the science. But you do this all the time Crakar (âAl Gore is getting rich! Al Gore is getting rich!â) so at least youâre staying predictable. You then added your own triumphant coda to these declarations of victory in your last post:

The balance of evidence suggests the theory of AGW is a crock of shit . . .

Unbelievable.

And this after telling us earlier that . . .

. . . i do not want to turn this into a lowbrow slugfest as some [i.e., Skip] would imply . . . â

All that being said, keep it up, mate.

I'll confess to not even realizing that the relationship was logged until you brought the subject up.

crakar

Enough of these idiotic links you post. I have urged you time after time after time after time to do a modicum of research, and to critically appraise the information you post here. And yet you continue to cut and paste fundamentally flawed and biased nonsense from denialist opinion sites.

Just so you are aware, I am making a list of sources that I will no longer respond to, other than to provide a simple statement of disapproval for your lack of intellectual rigour and inability to approach the issue from even the simplest level of credibility.

On the list so far are:

Joanne Nova
The Science and Public Policy Institute
Oregon Institute of Science and Medicine
Watts up with that

I will add to the list as appropriate.

How about you give us a real challenge and link to some science for a change, rather than just the uniformed opinions of others.

123

You spew forth utter bullshit and then at the last minute a moment of honesty which only highlights even more how much bullshit you speak.(sarcasm off)

How can you write such a post and then openly admit you have no idea what you are talking about?

Lets step through this CO2 issue a bit slower OK, we are told if CO2 goes up it will produce more heat which by itself is not a problem the problem occurs when this extra heats cause more water vapour which in turn causes GW.

Now look at the graph, and tell me how much additional heat will be produced by doubling CO2? Now think about how much this minimal heat will increase water vapour.

Apply a liberal amount of logic and common sense to the above.

crakar,

You do not understand the concept of feedback at all. (Have not looked at your graph, but I don't need to to identify your misunderstanding).

Let's follow your thought experiment through: CO2 doubles instantly, global temperature goes up the measly 1oC doubled CO2 can do on its own. Now you claim this will only cause a small increase in WV content of the atmosphere and thus a small extra increase in temperature. Case closed for you. But you are forgetting that WV responds again to that new temperature increase because you see time does not actually stop and start and freeze again as we are doing in our thought experiment. So we get additional warming as WV feeds back on itself.

Okay? Not doing the math here, as it has in fact been done many times by the experts and is presented in the IPCC reports, but I think that is a very straightforward explanation with simple understandable concepts and clearly shows you (and your source) are not seeing the whole picture at all.

A real challenge Mandas?

The link i posted cited all of the data from refutable sources. You deviously side step this by claiming you are not required to respond because it is on the list.

Congratulations you have just taken another step along the path of religious dogma.

Stunning.

The link i posted cited all of the data from refutable sources.

I'll forgive the Freudian slip, but let me ask you a simple, direct, question, Crakar:

Do you believe the logarithmic relationship between CO2 and heat shows that the AGW hypothesis is "a crock of shit" [your words].

Its a simple yes or no question as to whether you stand by your previous statement.

I will even preface my question by saying that

(1) I am laying a trap for you
(2) I predict you will dodge the question and refuse to answer it Yes or No.

So, I have stacked all the cards in your favor and even shown you my hand.

So I repeat my question as per above, and ask, What is your answer?

Thankyou for your response Coby and whilst i respect your views i do not agree with them.

Rather than freezing time lets freeze CO2 levels. Lets say at 20ppm the overall temp (from CO2 and feedbacks) is 1C for the sake of the argument.

Then we double to 40ppm the temp for CO2 is now 2C and the +ve FB makes it 2.5C.

Then we double again and again and again and each time the temp from CO2 goes up and the temp from +ve goes up....OK so far?

So here we are at 385ppm the average temp of the world is say 15C a small portion of that is from CO2 and the FB's to this make up another small portion.

Re this statement " But you are forgetting that WV responds again to that new temperature increase because you see time does not actually stop and start and freeze again as we are doing in our thought experiment. So we get additional warming as WV feeds back on itself."

Now if we could freeze the CO2 levels for the next 50 years would the FB component of warming go up or stay the same, judging by your post you are saying the system will continue to positively feed back on itself and the planet would continue to warm therefore we could not possibly freeze our CO2 levels could we because we would and in fact are now in runaway warming.

In fact this would be the case even before man invented fire. The sun would warm the Earth thus releasing more CO2 thus more WV etc and there (according to you) would be nothing that could stop this.

Based on this i find your thought experiment to be flawed.

Now back to what i was saying @385ppm if we increase the PPM until 770ppm then we will get 1C of warming plus a bit extra from WV, exactly how much depends on what you beleive to be the sensitivity etc.

Of course if we then double it to 1540ppm then we will get another 1C plus +ve FB's.

So as you can see the more CO2 that is released into the atmosphere the less effect it has regardless of Coby's radical hypothesis. If any of you bothered to look at the graph and i mean actually looked at the graph you will see that the effects of CO2 as in temp increases diminishes quite quickly.

Yet if i was to ask for evidence of the AGW hypothesis i would be presented with a couple of models.

The previous link i posted shows evidence of a number of things that have not changed (rainfall, snowfall etc) of course this is ignored in favour of unvalidated computer models and religious rantings.

I was just looking at the gucci shop i am after a pair of CFM boots for my wife, do you give a denier discount?

Will respond to your request a bit later Skip, keep an eye out.

He he.

Maybe we could go in together on a package. My wife hates Gucci but sea level rise might swamp all but Northern Italy so soon that I might miss out on my chance--and her birthday approaches.

Let me know . . . he he.

crakar

A science teacher in a high school is teaching biology and evolution, when a student at the back of the class loudly proclaims that evolution is a conspiracy, and it is completely false, because his preacher has told him that the bible is the unimpeachable word of god, and that therefore he will brook no argument which suggests the bible is flawed. The science teacher patiently lays out the case for evolution, including how it is consistent with every other science discipline and there are no other logical explanations for what we have observed. The student goes away, and comes back the next day with some information provided by his preacher, showing how all the fossil record is explained by a catastrophic worldwide flood that happened 5000 years ago. The science teacher patiently explains the flaws in the logic of the flood story, and shows how the earth is billions of years old, and how evolution is the only reasonably explanation of life on earth.

Once again, the student comes back the following day, and accuses the science teacher of heresy and loudly proclaims that evolution is heresy and is demonstrably false, because he has evidence that Darwin believed in god. Once again, the science teacher patiently explains that even if it were true that Darwin believed in god (which is debatable in any case), this in no way invalidates the science behind evolution. The science teacher once again explains how evolution works, and shows that evolution is not only a theory, but is actually occurring in living organisms and can be shown to be happening.

The student goes away, and comes back the next day with another quote from his preacher, stating that evolution is nonsense, because of irreducible complexity. The science teacher once again patiently explains how complex organs like an eye or a wing can evolve, and how even partial eyes and wings can be of benefit to an organism.

The student once again goes away, and comes back the next day and proclaims how his preacher had told him that evolution is false, because there are gaps in the fossil record that conclusively prove there are no transitional forms, and evolution is all a huge conspiracy to discredit the real truth, which is that god created man in his image, and that people who said otherwise were in the service of satan, and have ulterior motives.

The science teacher has finally had enough of this shit, and tells the student that he will no longer respond to the nonsensical statements of the preacher. They have been proven time and time again to be flawed, and the science teacher has only so much time and so much patience to respond to crap.

The student states that because of this, he has won, because the science teacher has obviously met his match and can not find any convincing arguments to support his obviously flawed opinions. God has won, and all is right in the world.

Sound familiar?

Thanks Mandas for that classroom story and yes it does sound familiar but not in the way you think. when my son was around 8 years old (thats your cue Ian) he loved dinosaurs and i had mistakenly sent him to a private religious school. When the teacher stated that God made the Earth and man in six days my son said thats impossible because the evidence shows dinosaurs were here 65 million years ago.

The teacher gob smacked by what he said recovered rather quickly ignored him and continued on. The differnce between the two stories is that yours is a fictional account of an event invented by you to try to prove a point whereas mine is based on fact.

For example the OLR leaving the atmosphere has stayed relatively stable even though CO2 emissions have been increasing at a rapid rate, Mandas please explain the errors in my link which shows this. Not a made up story by a story based on evidence.

Skip that sounds like a good idea let me know when you are going.

In regards to your stunning post 128.

I should firstly qualify what i mean by the AGW theory, that is the AGW theory is the one espoused by the IPCC.

So bearing this in mind i would have to yes the AGW theory is a crock.

If you asked my do i accept that increasing CO2 will add a slight warming then i would say no it is not a crock.

Did i spring your trap?

Of course not because--exactly as I predicted--

you did not answer the question.

I will paraphrase it in light of your reference to the IPCC

Do you believe the logarithmic relationship between CO2 and heat shows that the AGW hypothesis [as espoused by the IPCC] is "a crock of shit" [your words]?

It is a simple, yes or no question.

"For example the OLR leaving the atmosphere has stayed relatively stable even though CO2 emissions have been increasing at a rapid rate" - Crakar

Wrong again:

http://www.nature.com/nature/journal/v410/n6826/full/410355a0.html

"Here we analyse the difference between the spectra of the outgoing longwave radiation of the Earth as measured by orbiting spacecraft in 1970 and 1997. We find differences in the spectra that point to long-term changes in atmospheric CH4, CO2 and O3 as well as CFC-11 and CFC-12. Our results provide direct experimental evidence for a significant increase in the Earth's greenhouse effect that is consistent with concerns over radiative forcing of climate."

A few other papers on the topic here:

http://agwobserver.wordpress.com/2009/08/02/papers-on-changes-in-olr-du…

By Dappledwater (not verified) on 31 Mar 2010 #permalink

crakar

As your are in the RAAF, I am sure you are familiar with the phrase 'the bloody great wheel'. For those of you who are not familiar, it refers to the fact that if you stay in the same organisation long enough, things will keep changing til eventually they change back to the way they used to be in the past. People recognise this and refer to things going around and around like 'the bloody great wheel'.

Your post on OLR is just another example of that. I would like DW for providing a response to your idiocy - because had it been left to me alone I would simply have told you to fuck off, because we have already had this discussion, and provided exactly the same paper as DW (I actually provided an additional study from 2006 as well, but the point is well made in the 1997 paper.

This confirms my analogy about the science teacher and the creationist. You keep coming up with the same tired old cut-and-pastes from the deniershpere, and we keep shooting them down, only for you to find another, and another, till eventually you run out of new ideas and start rehashing old ones. Tell me, when will you start on about MWP and cosmic rays etc again? You haven't done them for a while.

Ah yes there is always another study that trumps the other isnt there DW.

Does your study differentiate between CO2, CH4 and O3? Does it also measure for water vapour?

Mandas,

Again with the language, you quite often pick up your bat and ball and go home when people disagree with you. I suspect you are a big sook.

The study i produced shows OLR to not have changed as much as the warmist would have liked so now it is no good and we scour the internet looking for snippetts in another that we can cling to.

No mention of snow fall, rainfall, global ice extent etc staying the same. Keep looking as i am sure you will find a vague reference to prop up your beliefs

"just as i predicted"

Are you using tarot cards or a crystal ball.

I thought i answered your simplistic question although i did use big words so let me rephrase it. The IPCC's hypothesis is a crock of shit.

Understand now.

Coby thanks for informing Ian he is an idiot on my behalf. By the way do you understand where you went wrong in post 126?

crakar

So, now that the main pillar of your support for the conspiracy theory - the hacked emails - has been eliminated by the findings of a parliamentary inquiry, are you going to act like an adult and admit you were wrong, or are you going to a) sulk and just ignore it, or b) accuse the parliamentary inquiry of a cover-up?

Are you using tarot cards or a crystal ball.

I am relying on your track record.

Proving once again that you are no dummy--as you've been accused of--and for which I've at times rushed to your defense, you have dodged my question as you obviously realize that answering it directly will corner you, Crakar.

Let me yet again re-ask my question, emphasizing the specific aspect of it that I want you to address.

Do you believe the logarithmic relationship between CO2 and heat shows that the AGW hypothesis [as espoused by the IPCC] is "a crock of shit" [your words]?

It is a simple question. It is a direct question. It lends itself to a simple yes or no answer.

Yes or no? It takes all of five seconds to answer.

Skip,

"Do you believe the logarithmic relationship between CO2 and heat shows that the AGW hypothesis [as espoused by the IPCC] is "a crock of shit" [your words]?"

I don't understand the reason for the question.
While the "logarithmic relationship" is a given and true, by itself, it neither proves or disproves the IPCC AGW hypothesis.

So, I suppose the "logical" answer is no.
Just as the answer is no for the "logarithmic relationship" proving the AGW hypothesis.

So, I suppose the "logical" answer is no.

At the moment, I'm not interested in the logical answer; I'm interested in *Crakar's* answer.

Just as the answer is no for the "logarithmic relationship" proving the AGW hypothesis.

It surely doesn't. Are you aware of anyone saying it does? I never did.

???
I'll continue to observe.

As will I.

Really, a slightly better formulation of important yes/no questions for Crakar go like this.

1. Do you believe that if there were no IR-absorbing gasses in our atmosphere at all, the temperature would be about 30C lower than it is? (A consequence of Stefan-Boltzman + albedo.)

2. Do you believe that the two most significant gases in raising the temperature above the S-B + albedo temperature are H2O and CO2?

3. Do you believe that H2O in the atmosphere is in relatively fast (a few days to a few weeks) equilibrium with the ocean and the ground? That is to say that while there are definitely seasonal patterns and weather "noise" to relative humidity, the amount of H2O in the atmosphere can't significantly change in the absence of some other forcing, and if the forcing is transient (say, ten million tea kettles pumping out steam somewhere for a few days) the response is transient.

That's it. That's the fundamental basis of AGW. If you answer no to any of those three questions, then you're wrong on the basic physics. If you answer yes, then while other forcings (solar, volcanic aerosols, etc.) can also cause warming or cooling, you must accept that (human or otherwise) alteration of the amount of CO2 in the atmosphere will have an effect on the temperature. From there it's just a matter of arguing about the sensitivity.

Oh, and Paul, I need to contextualize this discussion.

In 119 Crakar linked us to this graph and discussion of the logarithmic relationship between CO2 and heat.

http://wattsupwiththat.com/2010/03/08/the-logarithmic-effect-of-carbon-…

(This was back on 3/10. Ian quickly responded in 120, explaining that while the relationship is indeed logarithmic, the projected of increased temps by the IPCC is based *not* just on the independent effect of CO2, but the long term *positive feedback* of water vapor *resulting* from the increased heat.)

In 121, Crakar responded as follows:

Then the graph shows if we increase the CO2 level from say 385ppm to 770ppm there will be very little increase in heat so how could this possibly cause a much greater increase in WV so much so as to bring about the temp increases predicted by the IPCC?

No one responded for two weeks and Crakar became emboldened, writing in 122

I see we have successfully ignored the log relation ship of CO2, lets try this approach. . . So lets look at the balance of evidence. The following link will address the scary stories . . .

Craker then made a bizarre link to something about someone getting rich off global warming actionâa complete red herring, and triumphantly concluded

The balance of evidence suggests the theory of AGW is a crock of shit, anyone care to dispute this or will you all go very quiet as per the log graph i produced a couple of posts ago?

Crakarâs fundamental confusion, of course, is that heâs mistaking a lower *incremental* decrease in the warming effect of CO2 with a lower *absolute* warming effect of CO2 and its subsequent impact on water vapor as a positive feedback. Even though each additional increment of CO2 has less total effect on warming, its still more *total* warming and thus more *total* subsequent positive feedback, and the models are based on this cumulative process occurring *over time*.

I pointed out his mistake in 123, at which point Crakar disintegrated, telling me that I was

. . .spew[ing] forth utter bullshit . . .

and that my admission to having previously not known about the logarithmic function was proof that I was

admit[ing] that [I] have no idea what [I am] talking about . . .

He then repeated and betrayed his fundamental misunderstanding:

Now look at the graph, and tell me how much additional heat will be produced by doubling CO2? Now think about how much this minimal heat will increase water vapour.

Again, at the time, Crakar had somehow worked himself into a conviction that more CO2 means *less* heatâwhen again all it means is less *additional* heat per incremental increase of CO2. I suspect he knows his error now, but heâs probably a little embarrassed.

I know in some cultures its considered poor form to badger someone who has publicly humiliated himself and that we should graciously drop an uncomfortable subject to let him save face. But we're having a public discussion on a very important issue. Crakar's a big boy, and my question to him is fair even if there is no way he can answer it without looking silly one way or the other.

The paragraph above was poorly worded and should be read as follows:

"Crakarâs fundamental confusion, of course, is that heâs mistaking a diminishing *incremental* increase in the warming effect of CO2 with a lower *absolute* warming effect of CO2 and its subsequent impact on water vapor as a positive feedback. Even though each additional increment of CO2 has less additional effect on warming, its still increases the *total* warming and thus the *total* subsequent positive feedback, and the models are based on this cumulative process occurring *over time*."

Skip,
thanks for the further explanation.

I assumed the only continuing argument/discussion (including Crakar) is the identifications, strength and sign (+/-) of the feedbacks and other possible effects (cloud creation mechanisms, etc) and determining that reducing CO2 will have meaningful effect on temperature rise (or lowering) in the future.

Certainly Crakar has no disagreement that CO2 is a GHG, with lesser incremental effect as concentrations increase.

Well, I'll continue to observe, again thanks for the elaboration.

Certainly Crakar has no disagreement that CO2 is a GHG, with lesser incremental effect as concentrations increase.

Yeah but the crucial thing to recognize is that the water vapor feedback effect still *increases* as the the CO2 does. How that particular feedback is calculated is beyond me, I concede. But I'll take the IPCC's judgment before Crakar's.

Crakar seemed to want to add just the incremental temp increase and its feedback effect that you get at 700+ ppm, but you have to add all those other increments as well (Figuratively speaking, of course; the logged function is actually a downward curve and you add in essence, the area under the curve, but I'll let the math people do the calculus.)

Dont have a lot of time as the F1 qualifying is about to start and i need my laptop to watch it.

Skip 147 is a good summary but you seem a bit confused by what i am saying, i will accept that as sometimes i post in a hurry and this lends to the confusion. I think Paul has a good grip on what i am saying but i will elaborate later tonight.

cheers all
And com on Webber

Hey Crakar

Still waiting for your comments and apology for wasting our time regarding the hacked email scandal. Of course, you have read the report from the parliamentary inquiry - what do you think?